Given 4 1/10 -3 5/12 determine the product

Answers

Answer 1

The product of 4 1/10 and -3 5/12 is -14 1/120

What are fractions?

Fractions are simply described as a subset of a whole or the part of a whole.

They are several types of fractions, some of which includes;

Mixed fractionsProper fractionsImproper fractionsSimple fractionsComplex fractions

Some examples of mixed fractions; 3 1/4, 5 1/2, 6 1/4

Some examples of simple fractions; 1/2, 1/4, 1/6

Some examples of proper fractions; 3/4, 4/5, 6/7

Some examples of improper fractions; 3/2, 4/3, 6/5

Given the mixed fractions;

4 1/10  ×  -3 5/12

convert to improper fractions

41/10 × - 41/12

Multiply through

- 1681/120

Find the ratio

- 14 1/ 120

Hence, the value is - 14 1/ 120

Learn more about fractions here:

https://brainly.com/question/11562149

#SPJ1


Related Questions

Help please i forgot about this

Whats the product of 3 plus 4
whats 5 times 8

Answers

if you mean 3 times 4, then 3*4=12
5*8=40

John's commute to work is 20kmhr while Sheri's commute is 500mmin.


Who has the fastest commute to work in mihrif 1.61km=1mi?

Answers

By 6.2 miles per hour, Sheri commutes more quickly.

We are given that,

John travels to work = 20km/hr

Sheri travels to work = 500m/min converting this into km/hr we get ,

1.61km/hr or 1 mile

John travels to work at the following rate:

[tex]\frac{20 km}{1 hr} * \frac{1 mile}{1.61km} = 12.42 miles/hr[/tex]

Sheri travels work at the following rate:

[tex]\frac{500m}{1min} *\frac{1km}{1000 m}*\frac{1mile}{1.61km}*\frac{60min}{1hr}=18.63 miles/hr[/tex]

Computing the difference between the two,

18.63 miles/hr - 12.42 miles/hr = 6.21 miles/ hr ≈ 6.2 miles/ hr

thus, by 6.2 miles per hour, Sheri commutes more quickly.

What are conversions in maths?

An amount that is multiplied or divided between one set of units and another is known as a conversion factor. In the event that a conversion is necessary, it must be carried out with the proper conversion factor to produce an equivalent value. For instance, when translating between inches and feet, 12 inches equals one foot.

to learn more about conversions visit:

https://brainly.com/question/24376098

#SPJ9

35 = u/5 + 12 solve for u and simplify the answer as much as possible

Answers

Answer:

u = 115

Step-by-step explanation:

35 = u/5 + 12

35 - 12 = u/5

(35 - 12) * 5 = u

u = (35 - 12) * 5

u = 115

PLEASE HURRYYYY

The segment contains points S(a, b) and T(c, d). Which formula should be used to find the midpoint of the segment?

Answers

In this case, the formula you would use is (a+c)/2 , (b+d)/2.

In reality, you’re using the midpoint formula. The midpoint formula is as follows:

(x1+x2)/2 , (y1+y2)/2


For example,
If you had point (2,4) and (4,8) you would plug in the equation as follows.

(2,4) = x1,y1
(4,8) = x2,y2

(2+4)/2 , (4+8)/2
6/2 , 12/2
(3,6)

So the midpoint of the two points (2,4) and (4,8) would be (3,6).

The table shows the percent of each type of movie in Brayden's collection. What is the ratio of Action movies to all of the movies in Brayden's collection?



Movie Type Percent of Collection
Comedy 20%
Action 40%
Drama 15%
Science Fiction 25%


Select all that apply.

A.
2 : 5

B.
3
20
C.
2
3
D.
2
5
E.
3 : 20

Answers

the ratio of action movies to all movies is 2: 5 .

In mathematics, a ratio illustrates how frequently one number appears in another. On a fruit tray, the ratio would be eight to six if there were eight oranges and six lemons.

Ratios can be lowered by dividing each quantity by the sum of all the common factors between the numbers (much like fractions are).When the ratio's numbers are the smallest practical integers, the fraction is said to be in its simplest form.The ratio's antecedent and consequent are frequently referred to as A and B, respectively.

Let Brayden has x number of  movies in total.

Total number of action movies in  his collection = 40% of x = 0.4 x

Other movies = 1 - 0.4x = 0.6 x

Ratio of action movies to all movies = 0.4x : x = 2: 5

Therefore the ratio of action movies to all movies is 2: 5 .

To learn more about ratio visit:

brainly.com/question/13419413

#SPJ1

If g(x) = -x, for what value of x does
g(x) = -9?

Answers

Answer:

x = 9

Step-by-step explanation:

given g(x) = - x and g(x) = - 9 , the equate right sides

- x = - 9 ( multiply both sides by - 1 )

x = 9

PLEASE HELP I NEED THIS VERY VERY SOON!!!!

Tim and Jim leave Chicago traveling in opposite
directions. Jim travels 65 mph and Tim travels
63 mph. In how many hours will they be 1056
miles apart?

Answers

The formula for time, t = d/s which means time exists equivalent to distance divided by speed.

The times taken to travel 441 miles apart exists 8.25 hours

What is the time formula?

Time can be defined as the dimension based on which the evolution of any system takes place. It can be measured in terms of seconds, minutes, hours, days, weeks, months, and years.

The formula for time, t = d/s which means time exists equivalent to distance divided by speed.

Given: Train running in the opposite direction

Speed of one train (S1) = 65 mph

Speed of one train (S2)= 63 mph

Distance to be covered = 1056 miles

Train exists running in the opposite direction, so their relative speed = (S1 + S2)

Relative speed = (65 + 63) = 128 mph

Now, time is taken to travel 1056 miles apart

Time = Distance / relative speed

Time = 1056/128 = 8.25 hours

Therefore, the times taken to travel 441 miles apart exists 8.25 hours

To learn more about time refer to:

https://brainly.com/question/3004254

#SPJ13

Corinne made $60,400 last year. She received a 2% annual raise. What will her new salary be for the upcoming year?

Answers

Answer:

61608 i believe

Step-by-step explanation:

2% of 60400= 1208

60400+1208=61608

(X) 1.10.PS-7
Simplify the expression. Write the answer in scientific notation.
(7×10¯²) (9×10¯²)
(7×10¯²) (9×10¯²) = ¯
(Simplify your answer. Use scientific notation. Use the multiplication symbol in the r
Enter you

Answers

The scientific notation is 6.3 × [tex]10^{-3}[/tex]

The expression is ( 7 ×[tex]10^{-2}[/tex] )( 9 × [tex]10^{-2}[/tex])

Calculate the product or quotient

=63 × ([tex]10^{-2}[/tex] ×[tex]10^{-2}[/tex])

Simplify using the exponent rule with the same base [tex]a^{n} . a^{m}[/tex] = [tex]a^{n + m}[/tex]

=63 × [tex]10^{-2-2}[/tex]

=63 ×[tex]10^{-4}[/tex]

= 6.3 × [tex]10^{-3}[/tex]

Therefore the scientific notation is 6.3 ×[tex]10^{-3}[/tex].

To know more about the expression refer to the link given below:

https://brainly.com/question/13961297

#SPJ1

Evaluate the quantity of x cubed minus 2x squared plus 3x minus 7 end quantity divided by the quantity of x minus 1 end quantity period. x squared minus x plus 2 minus 5 divided by the quantity x minus 1 end quantity x squared minus 2x plus 2 minus 9 divided by the quantity x minus 1 end quantity x squared minus x plus 4 minus 9 divided by the quantity x minus 1 end quantity x cubed minus 2x plus 2 minus 5 divided by the quantity x minus 1 end quantity

Answers

Answer:

(x^3 + 3x^2 - 2x + 7)/x- 2

Expand the numerator in the above expression

(x^3 + 5x^2 - 2x^2 + 8x - 10x - 16 + 23)/(x - 2)

Rearrange the terms of the numerator in the above expression

(x^3 + 5x^2 + 8x - 2x^2 - 10x - 16 + 23)/(x- 2)

Factorize the numerator in the above expression

[x(x^2 + 5x + 8) - 2(x^2 + 5x + 8) + 23]/(x - 2)

Factor out x^2 + 5x + 8

[(x -2)(x^2 + 5x + 8) + 23]/(x - 2)

Split the fractions

(x -2)(x^2 + 5x + 8)/(x - 2) + 23/(x - 2)

Divide the common factors

(x^2 + 5x + 8) + 23/(x - 2) hope this help btw your welcome

Answer:

The answer is really option A. x^2 - x +2-5/x-1

Step-by-step explanation:

I just took the test and got it right :)

Pls answer the question on the image attatched.

Answers

Step-by-step explanation:

loga(x) calculates simply the exponent of a needed to get x as result.

so, 5 "to the power of the exponent of 5 needed to get 7" is 7 (we give 5 the exponent that it needs to create 7, so it creates 7).

the same thing with 2, where we give 2 the exponent it needs to create 1/49. so, it creates 1/49.

we get therefore

7 × 1/49 = 7/49 = 1/7

i give brainlest can somebdy pls help me

Answers

After evaluating the given fraction, the resultant answer is (C) 1¼.

What are fractions?An element of a whole is a fraction. The number is represented mathematically as a quotient, where the numerator and denominator are split. Both are integers in a simple fraction. A fraction appears in the numerator or denominator of a complex fraction. The numerator of a proper fraction is less than the denominator. There are three main categories of fractions in mathematics. Proper fractions, incorrect fractions, and mixed fractions are these three types. The expressions with a numerator and a denominator are called fractions.

So, 8 ÷ 6⅖:

Now, evaluate as follows

8 ÷ 6⅖8 ÷ 30+2/58 ÷ 32/58/1 × 5/325/4

Which can also be written as 1¼.

Therefore, after evaluating the given fraction, the resultant answer is (C) 1¼.

Know more about fractions here:

https://brainly.com/question/28933207

#SPJ13

The dash triangle is a dilation image of the solar triangle with a center at the origin. Is a dilation and enlargement or a reduction find the scale factor of the dilation.

Answers

Because the scaling factor is a whole number, which is 3, the enlargement of the dashed triangle is a dilatation.

Recall:

Dilation enlarges or shrinks a figure or shape.The original image prior to any reduction or expansion is the image of a dilatation.The new image created following any necessary reduction or enlargement is the scale copy.Any side length of the new image divided by the corresponding side length of the old image gives the scale factor.The dilation is a reduction if the scale factor is a fraction.The dilatation is an enlargement if the scale factor is a whole number.

Thus:

For the dashed triangle, measure the separation between the points (2, -2) and (0, -2).

(2, -2) to (0, -2) is separated by 2 units.

Using the coordinates (6, -6) and (0, -6) for the solid triangle, calculate the equivalent distance:

(6, -6) and (0, -6) are separated by 6 units.

Scale factor = 6/2 = 3

The dilation is an enlargement because the scale factor is a whole number.

In conclusion, the enlarged dashed triangle indicates a dilatation because the scale factor, which is 3, is a whole number.

To know more about Dilation, refer to this link:

https://brainly.com/question/16727612

#SPJ1

A robot can complete 5 tasks in 3/4 hour. each task is the same. how long does it take the robot to complete one task

Answers

The robot will take 0.15 hours to complete one task.

What is the unitary method of problem solving?
A single unit's value can be determined from the values of multiple units, and multiple units' values can be determined from the values of single units using the unitary technique. The unitary technique involves first determining the value of a single unit, followed by the value of the necessary number of units. In the unitary method, the value of a unit quantity is determined before the values of other units are determined. Direct variation and inverse variation are its two different forms of variations. When there is a direct variation, an increase or decrease in one quantity will result in an equivalent rise or fall in the other. If we increase one quantity in inverse variation, the value of another quantity will drop.

Given, time taken by robot to complete five tasks = 0.75 hours
Thus, time taken by robot to complete one task = (0.75/5) = 0.15 hours
Therefore, the robot will take 0.15 hours to complete one task.

To learn more about this, tap on the link below:
https://brainly.com/question/24587372

#SPJ9

is 12x17=6x34 true or is it flase i cant do it

Answers

The answer is that it is true
True because 12x17 is 204 and 6x34 is also 204
204=204

A college has 19 engineering students for every 81 students enrolled. if the college has 750 students, approximately how many are engineering students

Answers

The total number of engineering students is 176 approximately among the 750 students enrolled in the college by using the proportionate formula.

It is mentioned here that a college has 19 students of engineering for every 81 students who got enrolled.

Therefore the ratio of engineering students to total number of students is as follows:

No. of engineering students = 19

Total number of students = 81

Ratio = [tex]\frac{19}{81}[/tex]

Now if the total number of students that the college has is 750 then the number of engineering students can be found by using the proportionate formula as the ratio of the engineering to the total students will remain same.

Total number of students = 750

Let the number of engineering students be y

Ratio = [tex]\frac{y}{750}[/tex]

Comparing it with the above ratio and using the proportionate formula,

[tex]\frac{19}{81} = \frac{y}{750}[/tex]

⇒ y = [tex]\frac{19*750}{81}[/tex]

⇒ y = 175.925

So approximately 176 were the engineering students among 750 students enrolled.

To learn more about proportionate formula click here:

https://brainly.ph/question/1455597

#SPJ4

Question 12
REASONING Given: 2a²
a
= 72. Conjecture: a = 6. Write a counterexample.

Answers

A counterexample to show that 2a^2 = 72 where a = 6 is that a = -6 will still arrive at 72

What is counterexample?

This is a term that counters a statement. we can therefore say that counterexample is one that proves a statement to be false.

How to write the counterexample

given data

2a² = 72

Conjecture: a = 6

The counterexample is a = -6

substituting a = -6 into the given equation

= 2a²

= 2 * ( -6 )^2

= 2 * ( 36 )

= 72

Since a = -6 gave same answer of 72 as a = 6 then a = -6 is the counterexample

Read more on counterexample here: https://brainly.com/question/5187310

#SPJ1

Two pools are being filled with water. To start, the first pool had 728 liters of water and the second pool was empty. Water is being added to the first pool at a rate of 23 liters per minute. Water is being added to the second pool at a rate of 37 liters per minute.
Let x be the number of minutes water has been added.

(a)
For each pool, write an expression for the amount of water in the pool after x minutes.
Amount of water in the first pool (in liters) = ____
Amount of water in the second pool (in liters) = ____


(b)
Write an equation to show when the two pools would have the same amount of water.

______________

Answers

Two pools are being filled with water. To start, the first pool had 728 liters of water and the second pool was empty. Water is being added to the first pool at a rate of 23 liters per minute. Water is being added to the second pool at a rate of 37 liters per minute.

Let x be the number of minutes water has been added.

(a)

For each pool, write an expression for the amount of water in the pool after x minutes.

Amount of water in the first pool (in liters) = 23x + 728

Amount of water in the second pool (in liters) = 37x

(b)

Write an equation to show when the two pools would have the same amount of water.

this is when:

23x + 728 = 37x

subtract 23x from both sides:

23x + 728 - 23x = 37x - 23x

728 = 14x

divide both sides by 14:

728/14 = 14x/14

x = 52

So, pools are equal after 52 minutes.

how do i multiply these ? (5x-6.9)(2x+12.2)

Answers

Follow the pemdas rule Parenthesis exponents multiplication division addition subtraction

Answer:

10x^2 + 47.2x - 84.18

Step-by-step explanation:

You distribute both terms in the first set of parentheses (5x-6.9) to the terms in the second set (2x+12.2) So when you're doing it 5x gets multiplied by 2x and 12.2 and -6.9 gets multiplied by 2x and 12.2. Then you simplify as much as possible

On your own sheet of paper, use long division to find the quotient of 5,550 ÷ 39. What is the remainder?

Answers

Answer: Quotient 142

Remainder 12

Step-by-step explanation:

The quotient is 142 and  the remainder is 12.

The long division method is as follows :

                                             [tex]142[/tex]

                                      [tex]39\sqrt{5550}[/tex]  

                                          -39                        

                                             165                       (3rd 5 taken down )

                                           -156    

                                                 90                     ( 0 taken down )

                                                -78  

                                                  12

step 1 : take the first two digits (55) of the divident , now multiply 39 with such a number that the product of those two must not go above 55 but should be closest to it.

step 2 : write the number on top and subtract the answer from 55.

step 3 :  Now bring down the third 5 and write it after the answer of the subtraction. So the number becomes 165.

step 4 : Again multiply 39 with such a number that the product of those two must not go above 165 but should be closest to it.

step 5 : write that number on top after the previous number and subtract the answer from 165.

step 6 : Now bring down the 0 and write it after the answer of the subtraction. So the number becomes 90.

step 7 : Again multiply 39 with such a number that the product of those two must not go above 90 but should be closest to it.

step 8 : write that number on top after the previous number and subtract the answer from 90.

We have to repeat these steps until all the digits from the divident are used and the remainder comes out to less than the divisor.

Here all the digits of divident are used and the remainder (12) is also less than the divisor (39), so we stopped.

Hence the quotient is 142 and the remainder is 12.

learn about long division method here :

https://brainly.com/question/3935316

#SPJ4

3.if the m∠1 is 57° , find the measure of ∠6.
Show your work

Answers

123°

180°-57°=<6=123°

Guys i need help is this right?​

Answers

The range of the function in this situation as required in the task content is; Choice D; 0 ≤ y ≤ 3.

How to determine the range of a function?

It follows from the task content that the range of the function described by the situation is to be determined.

The set of all possible output values of a function, otherwise termed the dependent variable, y is called the Range.

It follows that since the distance he has remaining, y is a function of how long he has been riding the bike, x and it's said that; the boy lives 3 miles from the library; the range of the function described is; Choice D ; 0 ≤ y ≤ 3.

Please Note: The term, domain encompasses the set of all possible input values, otherwise termed the independent variable, x.

Read more on range of a function;

https://brainly.com/question/7644636

#SPJ1

What is the
Range??
Help me out plsss

Answers

Answer:

1≤x≤|R+

Step-by-step explanation:

since range is from the y axis

the starting point is 1 UpTo positive real numbers and it doesn't include 0

In a Sports league each team has 36 players and 3 coaches. There are also a number of team assistants. The ratio of team assistants to players is 1/6. What is the ratio of coaches to assistants?

Answers

Answer:

1/2

Step-by-step explanation:

We know that each team has the following:

36Players + 3Coaches + ?Assistants = Sports League

We know that the ratio of assistants to players is 1/6, rewrite the equation accordingly:

36Players + 3Coaches + 6Assitants = Sports League

Now find the ratio of coaches to assistants:

3Coaches / 6Assistants = 1/2

What number does
2(x+3)²
3(x-1)²
represent WHEN x = 3?

Answers

Answer:

72 and 12

Step-by-step explanation:

substitute x = 3 into the expressions

2(x + 3)²

= 2(3 + 3)²

= 2(6)²

= 2 × 36

= 72

--------------------

3(x - 1)²

= 3(3 - 1)²

= 3(2)²

= 3 × 4

= 12

Which recursive sequence would sequence 5,-8,5

Answers

The recursive sequence that defines 5, -8, 5, is given by:

[tex]a_n = -a_{n-1} - 3[/tex]

In which the first term is of [tex]a_1 = 5[/tex].

What is a recursive sequence?

A recursive sequence, in general terms, can be defined as follows:

[tex]a_n = f(a_{n - 1))[/tex]

In which the first term [tex]a_1[/tex] is given.

This rule means that the nth term of the sequence is a function depending on the previous term, the (n-1)th term of the function.

In the context of this problem, the sequence is given as follows:

5, -8, 5.

From the first to the second term, the number changed the signal and was subtracted by 3, and the same is true for the second to third term, as follows:

-5 - 3 = -8.-(-8) - 3 = 8 - 3 = 5.

Then the recursive rule for the sequence is given as follows:

[tex]a_n = -a_{n-1} - 3[/tex]

With first term of [tex]a_1 = 5[/tex].

More can be learned about recursive sequences at https://brainly.com/question/27868858

#SPJ1

Consider the matrices.

What matrix results from B + A?

Enter your answer by filling in the boxes.

Answers

Answer:

Step-by-step explanation:

[tex]\displaystyle\\A=\left[\begin{array}{ccc}-1&6\\-2&2\\4&9\\-5&-11\end{array}\right] \ \ \ \ \ \ \ \ \ B=\left[\begin{array}{ccc}-4&-1\\-8&8\\2&-7\\-6&5\end{array}\right] \\\\A+B=B+A\ (commutativity)\\\\\Rightarrow B+A=\left[\begin{array}{ccc}-4+(-1)&-1+6\\-8+(-2)&8+2\\2+4&-7+9\\-6+(-5)&5+(-11)\end{array}\right] \\\\\\ \Rightarrow B+A=\left[\begin{array}{ccc}-5&5\\-10&10\\6&2\\-11&-6\end{array}\right][/tex]

A straight road makes an angle of 15 degrees with the horizontal. When the angle of elevation of the sun is 57 degrees, a vertical pole at the side of the road casts a shadow 75 feet long directly down the road, as shown in the figure. Approximate the length of the pole. Round to the nearest hundredth.

Answers

The length of the pole is 19.15 feet.

What is the angle of elevation?

The angle of elevation is the angle created when an observer looks at an object placed above its height in relation to the eye level or the horizontal line. An angle of elevation is, for example, the angle created between the line of sight and the horizontal line when a man on Earth observes the Sun.

Given:

Angle made by straight road = 15°

Angle of elevation of the sun = 57°

Length of the shadow = 75 feet

∠ACB = 90° - 57° = 33°

∠CAB = 57° - 15° = 42°

Using Sine Law to find BC i.e.length of the pole.

[tex]\frac{BC}{sin A} = \frac{AB}{sin C}[/tex]

[tex]\frac{BC}{sin 42} = \frac{75}{sin 33}[/tex]

[tex]BC= \frac{75sin 42}{sin 33}[/tex]

BC ≅ 19.15 feet (to the nearest hundredth.)

Hence, the approximate length of the pole is 19.15 feet.

To know more about angle of elevation visit:

https://brainly.com/question/16716174

#SPJ10

F(x)=3x^2-4x+8G(x)=2x-5 use the functions above to find the value of g(f(-2)) I don’t understand how to do this

Answers

Given,

[tex]f(x)=3x^2-4x+8,\text{g(x)=2x-5}[/tex]

To find g(f(-2)). we first find the value of f(-2).

The value of f(-2) is,

[tex]\begin{gathered} f(-2)=3(-2)^2-4(-2)+8 \\ \text{ =3(4)+8+8} \\ \text{ =12+8+8} \\ \text{ =28} \end{gathered}[/tex]

So now to find the value of g(f(-2)),

[tex]g(f(-2))=g(28)[/tex]

So the value of g(28) is,

[tex]\begin{gathered} g(28)=2(28)-5 \\ \text{ =56-5} \\ \text{ =51} \end{gathered}[/tex]

So the value of g(f(-2)) is 51.

[3^2 x (189 divided by 9] -30

Answers

Step-by-step explanation:

[tex]3 {}^{2} (189 \div 9) - 30 \\ 9 \times 189 \div 9 - 30 \\ 189 \div 1 - 30 \div 1 \\ 189 - 30 \div 1 \\ = 159[/tex]

according to bodmas

Answer:

159

Step-by-step explanation:

Using BODMAS - Bracket Of Division Multiplication Addition Subtraction

Step 1: Bracket

189/9=21

Hence [3^2 x (21)]-30

Step 2:

3^2= 9

Step 3: Calculate within parentheses [9* 21] :    189

9*21=189

Step 4: Open bracket

[189]-30=159

Other Questions
Given f(x) = x3 2x2 x + 2, the roots of f(x) are 1, 1, 2 . Determine the end behavior of f(x). The length of a base of a rectangular prism is 12 dm and the width is 5 dm. What should be the height of the rectangular prism be so that it's volume is smaller than the volume of a cube with a side of 9 dm? (for 30 points) will give you brainlyiest or smthing title Calculate the mass of 4.00 moles of NHOH (s). Which of the following best describes the location of the kidneys Identify and write the claim Douglass makes. describe your own experience with the contrast between society's trends toward globalism or nationalism over the past 10 years. what are the implications of these shifts for international businesses operating in the united states? in november, trinity donuts incurred $8850 in total costs related to the production of 3500 donuts. in december, trinity incurred $8600 in total costs. total fixed costs are $1850 per month. by how many donuts did the production change? (answer as a negative number if production has decreased). a cooler at picnic contain ASAP PLEASE HURRYWhat is the value of q 8 if q = 18?A. 26B. -26C. 10D. -10 A rectangle is 32ft by 8ft. Stephanie dilated the rectangle and the dimensions of the image were 24ft by 6ft. What was the scale factor used? In the planning stage of writing your essay, which question should you answer first:How many paragraphs do I need to write?What can I use for my hook?How many words does the essay need to be?What is the prompt asking me to address? It just barely missed me, but in my place it swallowed everything that mattered most to me and swept it off to another world." asymbolism bsimile cmetaphor dpersonification Triangle ABC is shown in this figure. 2 A B Which statement justifies that the sum of Angle 1 and Angle 2 is 180? O All adjacent angles are supplementary. O Adjacent angles that form a linear pair are supplementary. O The sum of any interior and any exterior angle of a triangle is 180 0 Each pair of interior and exterior angles that lie on a line is complementary What is the most precise name for quadrilateral ABCD with verticesA(-5, 7), B(6,-3), C(10, 2), and D(-1, 12)?A. rectangleB. rhombusC. squareD. parallelogram Question content area topPart 1A box of cereal states that there are calories in a -cup serving. What is the unit rate for calories per cup? How many calories are there in of the cereal? A Curve has the equation y=kx^2 +1 And a line has the equation y=kx whereK is anon-zeroConstant1)Find the Setthe value of kfor which thecurve and line have no common point.(1) state the value for which the line is atangent to the Curve and for this case, find the coordinates of the point where the linetouches the curves The concession stand are the football games are 288 customers. Those customers, about 77% bought a hot dog. About how many customers buy a hotdog? whats the correct answer answer asap for brainlist Solve for s.19.1 = s 7.3s = Simple Interest Round answers to the nearest cent. 1. John is going to invest $100 in an account for 3 years. If the account earns 6% simple interest, how much interest can he expect to earn?